Convergence/Divergence of infinite series $sum_{n=1}^{infty} frac{(sin(n)+2)^n}{n3^n}$












46












$begingroup$


$$ sum_{n=1}^{infty} frac{(sin(n)+2)^n}{n3^n}$$



Does it converge or diverge? Can we have a rigorous proof that is not probabilistic? For reference, this question is supposedly a mix of real analysis and calculus.










share|cite|improve this question











$endgroup$








  • 6




    $begingroup$
    Presumably the answer has to do with how well $pi$ can be approximated by rationals.
    $endgroup$
    – GEdgar
    Feb 14 '12 at 1:36






  • 10




    $begingroup$
    Also related: $sum_{n=1}^infty frac{|sin(n t)|^n}{n}$ converges for almost every real $t$ (in the sense of Lebesgue measure), but diverges for $t$ in a dense $G_delta$ subset of $mathbb R$ (thus for "generic" $t$ in the sense of Baire category). Which case is $t=1$ in? Nobody knows (but my bet would be on Lebesgue)
    $endgroup$
    – Robert Israel
    Feb 16 '12 at 2:37








  • 5




    $begingroup$
    @RobertIsrael: Really? Nobody knows whether $sum |sin n|^n/n$ converges?? Crazy, man, crazy!
    $endgroup$
    – Ben Crowell
    Feb 16 '12 at 3:16








  • 5




    $begingroup$
    @Robert: That's very interesting. Do you have a reference for that?
    $endgroup$
    – joriki
    Feb 16 '12 at 3:22






  • 4




    $begingroup$
    Wow, this question has turned into a graveyard of flawed answers -- four have been deleted so far.
    $endgroup$
    – joriki
    Feb 16 '12 at 14:18


















46












$begingroup$


$$ sum_{n=1}^{infty} frac{(sin(n)+2)^n}{n3^n}$$



Does it converge or diverge? Can we have a rigorous proof that is not probabilistic? For reference, this question is supposedly a mix of real analysis and calculus.










share|cite|improve this question











$endgroup$








  • 6




    $begingroup$
    Presumably the answer has to do with how well $pi$ can be approximated by rationals.
    $endgroup$
    – GEdgar
    Feb 14 '12 at 1:36






  • 10




    $begingroup$
    Also related: $sum_{n=1}^infty frac{|sin(n t)|^n}{n}$ converges for almost every real $t$ (in the sense of Lebesgue measure), but diverges for $t$ in a dense $G_delta$ subset of $mathbb R$ (thus for "generic" $t$ in the sense of Baire category). Which case is $t=1$ in? Nobody knows (but my bet would be on Lebesgue)
    $endgroup$
    – Robert Israel
    Feb 16 '12 at 2:37








  • 5




    $begingroup$
    @RobertIsrael: Really? Nobody knows whether $sum |sin n|^n/n$ converges?? Crazy, man, crazy!
    $endgroup$
    – Ben Crowell
    Feb 16 '12 at 3:16








  • 5




    $begingroup$
    @Robert: That's very interesting. Do you have a reference for that?
    $endgroup$
    – joriki
    Feb 16 '12 at 3:22






  • 4




    $begingroup$
    Wow, this question has turned into a graveyard of flawed answers -- four have been deleted so far.
    $endgroup$
    – joriki
    Feb 16 '12 at 14:18
















46












46








46


21



$begingroup$


$$ sum_{n=1}^{infty} frac{(sin(n)+2)^n}{n3^n}$$



Does it converge or diverge? Can we have a rigorous proof that is not probabilistic? For reference, this question is supposedly a mix of real analysis and calculus.










share|cite|improve this question











$endgroup$




$$ sum_{n=1}^{infty} frac{(sin(n)+2)^n}{n3^n}$$



Does it converge or diverge? Can we have a rigorous proof that is not probabilistic? For reference, this question is supposedly a mix of real analysis and calculus.







calculus real-analysis sequences-and-series






share|cite|improve this question















share|cite|improve this question













share|cite|improve this question




share|cite|improve this question








edited Feb 13 '12 at 21:08







user21436

















asked Feb 13 '12 at 20:59









badreferencesbadreferences

42147




42147








  • 6




    $begingroup$
    Presumably the answer has to do with how well $pi$ can be approximated by rationals.
    $endgroup$
    – GEdgar
    Feb 14 '12 at 1:36






  • 10




    $begingroup$
    Also related: $sum_{n=1}^infty frac{|sin(n t)|^n}{n}$ converges for almost every real $t$ (in the sense of Lebesgue measure), but diverges for $t$ in a dense $G_delta$ subset of $mathbb R$ (thus for "generic" $t$ in the sense of Baire category). Which case is $t=1$ in? Nobody knows (but my bet would be on Lebesgue)
    $endgroup$
    – Robert Israel
    Feb 16 '12 at 2:37








  • 5




    $begingroup$
    @RobertIsrael: Really? Nobody knows whether $sum |sin n|^n/n$ converges?? Crazy, man, crazy!
    $endgroup$
    – Ben Crowell
    Feb 16 '12 at 3:16








  • 5




    $begingroup$
    @Robert: That's very interesting. Do you have a reference for that?
    $endgroup$
    – joriki
    Feb 16 '12 at 3:22






  • 4




    $begingroup$
    Wow, this question has turned into a graveyard of flawed answers -- four have been deleted so far.
    $endgroup$
    – joriki
    Feb 16 '12 at 14:18
















  • 6




    $begingroup$
    Presumably the answer has to do with how well $pi$ can be approximated by rationals.
    $endgroup$
    – GEdgar
    Feb 14 '12 at 1:36






  • 10




    $begingroup$
    Also related: $sum_{n=1}^infty frac{|sin(n t)|^n}{n}$ converges for almost every real $t$ (in the sense of Lebesgue measure), but diverges for $t$ in a dense $G_delta$ subset of $mathbb R$ (thus for "generic" $t$ in the sense of Baire category). Which case is $t=1$ in? Nobody knows (but my bet would be on Lebesgue)
    $endgroup$
    – Robert Israel
    Feb 16 '12 at 2:37








  • 5




    $begingroup$
    @RobertIsrael: Really? Nobody knows whether $sum |sin n|^n/n$ converges?? Crazy, man, crazy!
    $endgroup$
    – Ben Crowell
    Feb 16 '12 at 3:16








  • 5




    $begingroup$
    @Robert: That's very interesting. Do you have a reference for that?
    $endgroup$
    – joriki
    Feb 16 '12 at 3:22






  • 4




    $begingroup$
    Wow, this question has turned into a graveyard of flawed answers -- four have been deleted so far.
    $endgroup$
    – joriki
    Feb 16 '12 at 14:18










6




6




$begingroup$
Presumably the answer has to do with how well $pi$ can be approximated by rationals.
$endgroup$
– GEdgar
Feb 14 '12 at 1:36




$begingroup$
Presumably the answer has to do with how well $pi$ can be approximated by rationals.
$endgroup$
– GEdgar
Feb 14 '12 at 1:36




10




10




$begingroup$
Also related: $sum_{n=1}^infty frac{|sin(n t)|^n}{n}$ converges for almost every real $t$ (in the sense of Lebesgue measure), but diverges for $t$ in a dense $G_delta$ subset of $mathbb R$ (thus for "generic" $t$ in the sense of Baire category). Which case is $t=1$ in? Nobody knows (but my bet would be on Lebesgue)
$endgroup$
– Robert Israel
Feb 16 '12 at 2:37






$begingroup$
Also related: $sum_{n=1}^infty frac{|sin(n t)|^n}{n}$ converges for almost every real $t$ (in the sense of Lebesgue measure), but diverges for $t$ in a dense $G_delta$ subset of $mathbb R$ (thus for "generic" $t$ in the sense of Baire category). Which case is $t=1$ in? Nobody knows (but my bet would be on Lebesgue)
$endgroup$
– Robert Israel
Feb 16 '12 at 2:37






5




5




$begingroup$
@RobertIsrael: Really? Nobody knows whether $sum |sin n|^n/n$ converges?? Crazy, man, crazy!
$endgroup$
– Ben Crowell
Feb 16 '12 at 3:16






$begingroup$
@RobertIsrael: Really? Nobody knows whether $sum |sin n|^n/n$ converges?? Crazy, man, crazy!
$endgroup$
– Ben Crowell
Feb 16 '12 at 3:16






5




5




$begingroup$
@Robert: That's very interesting. Do you have a reference for that?
$endgroup$
– joriki
Feb 16 '12 at 3:22




$begingroup$
@Robert: That's very interesting. Do you have a reference for that?
$endgroup$
– joriki
Feb 16 '12 at 3:22




4




4




$begingroup$
Wow, this question has turned into a graveyard of flawed answers -- four have been deleted so far.
$endgroup$
– joriki
Feb 16 '12 at 14:18






$begingroup$
Wow, this question has turned into a graveyard of flawed answers -- four have been deleted so far.
$endgroup$
– joriki
Feb 16 '12 at 14:18












5 Answers
5






active

oldest

votes


















28





+100







$begingroup$

The values for which $sin(n)$ is close to $1$ (say in an interval $[1-varepsilon ; 1]$) are somewhat regular :



$1 - varepsilon le sin(n)$ implies that there exists an integer $k(n)$ such that
$n = 2k(n) pi + frac pi 2 + a(n)$ where $|a(n)| leq arccos(1- varepsilon)$.
As $varepsilon to 0$, $arccos(1- varepsilon) sim sqrt{2 varepsilon}$, thus
we can safely say that for $varepsilon$ small enough, $|n-2k(n) pi - frac{pi}2| = |a(n)| leq 2 sqrt{ varepsilon} $



If $m gt n$ and $sin(n)$ and $sin(m)$ are both in $[1-varepsilon ; 1]$,
then we have the inequality $|(m-n) - 2(k(m)-k(n)) pi| leq |m-2k(m)pi - frac{pi}2| + |n-2k(n)pi - frac{pi}2| leq 4 sqrt { varepsilon} $ where $(k(m)-k(n))$ is some integer $k$.



Since $pi$ has a finite irrationality measure, we know that there is a finite real constant $mu gt 2$ such that for any integers $n,k$ large enough,
$|n-k pi| ge k^{1- mu} $.



By picking $varepsilon$ small enough we can forget about the finite number of exceptions to the inequality, and we get $ 4sqrt{varepsilon} ge (2k)^{1- mu}$.
Thus $(m-n) ge 2kpi - 4sqrt{varepsilon} ge pi(4sqrt{varepsilon})^{frac1{1- mu}} - 4sqrt{varepsilon} ge A_varepsilon = Asqrt{varepsilon}^{frac1{1- mu}} $ for some constant $A$.



Therefore, we have a guarantee on the lengh of the gaps between equally problematic terms, and we know how this length grows as $varepsilon$ gets smaller (as we look for more problematic terms)



We can get a lower bound for the first problematic term using the irrationality measure as well : from $|n-2k(n) pi - frac{pi}2| leq 2sqrt {varepsilon}$, we get that for $varepsilon$ small enough, $(4k+1)^{1- mu} le |2n - (4k+1) pi| le 4sqrt varepsilon$, and then $n ge B_varepsilon = Bsqrtvarepsilon^{frac1{1- mu}}$ for some constant $B$.



Therefore, there exists a constant $C$ such that forall $varepsilon$ small enough, the $k$-th integer $n$ such that $1-varepsilon le sin n$ is greater than $C_varepsilon k = Csqrtvarepsilon^{frac1{1- mu}}k$



Since $varepsilon < 1$ and $frac 1 {1- mu} < 0$, this bound $C_ varepsilon$ grows when $varepsilon$ gets smaller.
And furthermore, the speed of this growth is greater if we can pick a smaller (better) value for $mu$ (though all that matters is that $mu$ is finite)





Now let us give an upper bound on the contribution of the terms where $n$ is an integer such that $sin (n) in [1-2varepsilon ; 1-varepsilon]$



$$S_varepsilon = sum frac{(2+sin(n))^n}{n3^n} le sum_{kge 1} frac{(1- varepsilon/3)^{kC_{2varepsilon}}}{kC_{2varepsilon}} = frac{- log (1- (1- varepsilon/3)^{C_{2varepsilon}})}{C_{2varepsilon}} \
le frac{- log (1- (1- C_{2varepsilon} varepsilon/3))}{C_{2varepsilon}}
= frac{- log (C_{2varepsilon} varepsilon/3))}{C_{2varepsilon}}
$$



$C_{2varepsilon} = C sqrt{2varepsilon}^frac 1 {1- mu} = C' varepsilon^nu$ with $ nu = frac 1 {2(1- mu)} in ] -1/2 ; 0[$, so :



$$ S_varepsilon le - frac{ log (C'/3) + (1+ nu) log varepsilon}{C'varepsilon^nu}
$$





Finally, we have to check if the series $sum S_{2^{-k}}$ converges or not :



$$ sum S_{2^{-k}} le sum - frac { log (C'/3) - k(1+ nu) log 2}{C' 2^{-knu}}
= sum (A+Bk)(2^ nu)^k $$



Since $2^ nu < 1$, the series converges.






share|cite|improve this answer











$endgroup$













  • $begingroup$
    @ bgins : I don't think so. In order to recognize a Lebesgue-type thing, from what I remember I would have to sort the terms according to the value of ((2+sin(n))/3)^n and not simply sin(n), and it would be harder to evaluate the corresponding contribution. Maybe someone more knowledgeable can answer about this.
    $endgroup$
    – mercio
    Feb 16 '12 at 17:29






  • 1




    $begingroup$
    +1, very nice. I started down the same $sin n in [1-2varepsilon ; 1-varepsilon]$ route but didn't immediately see how to derive the required bounds that you derive in the first part -- I didn't pursue it any further when I saw Robert's comment about $sum_{n=1}^infty frac{|sin(n t)|^n}{n}$. Doesn't this proof also apply to that series for $t=1$? It seems the only differences are the factor of $1/3$ and the absolute value, but neither should matter. (By the way I think it should be $muge2$ instead of $mugt2$?)
    $endgroup$
    – joriki
    Feb 16 '12 at 18:02








  • 1




    $begingroup$
    @joriki : I think the proof applies to $sum frac{|sin(nt)|^n}n$ for any $t$ where the irrationality measure of $frac t pi$ is finite. mathworld says Salikhov proved his result in 2008, I don't know if the irrationality measure of $pi$ was proven to be finite before then or not. About $mu$, originally I wanted to be on the safe side and thought of it as a number strictly greater than the irrationality measure (which is an infimum, maybe not a minimum...). Maybe we can have $mu = 2$ in some cases but it doesn't matter ultimately.
    $endgroup$
    – mercio
    Feb 16 '12 at 18:41






  • 1




    $begingroup$
    mercio: Right. Then the appeal to Salikhov might be debatable, since the finiteness of $mu$ was known before Salikhov's explicit upper bound (am I right if I say this goes back at least (and maybe exactly) to Mahler 1953 with an estimate like $muleqslant20$?).
    $endgroup$
    – Did
    Feb 17 '12 at 12:54






  • 2




    $begingroup$
    mercio: If I may insist: hence one cannot write that Salikhov proved that $pi$ has a finite irrationality measure (fourth paragraph).
    $endgroup$
    – Did
    Feb 17 '12 at 15:10



















12












$begingroup$

I knew this looked familiar. User Unoqualunque located the key reference. Here is a more recent reference that provides a fairly general approach:




Enrico Laeng, Vittorino Pata. A convergence–divergence test for series of nonnegative
terms
, Expositiones Mathematicae 29 (4), (2011) 420–424. MR2861768 (2012m:40002).




The authors discuss a test that does not require monotonicity of the sequence, and instead focuses on how "clustered" we find similar terms within the sequence.



They highlight that their test applies to show that
$$ sum_{n=1}^inftyfrac1{n^{2+cos n}} $$
diverges, while
$$ sum_{n=1}^inftyfrac1nleft(frac{2+cos n}3right)^n $$
converges. They say:




Case (i) has been recently addressed, in (Revue de la filière Mathématique (RMS) 119 (2008–2009), 3–8), where the authors give a proof that was (in their own words) at the frontier between analysis and number theory. Case (ii) apparently originated in a curious way: it was proposed in a calculus exam by mistake, and remained open for a long time thereafter. A solution was devised only ten years later (SIAM Problems and Solutions (2009)), once again by means of quite sophisticated tools.




As is to be expected, the test is very general but a bit cumbersome to state:



Let $(c_n)_{nge1}$ be a sequence of nonnegative terms such that $sum_n c_n<+infty$. Let $(a_n)_{nge1}$ be a series of nonnegative terms. Then:




  1. The series $sum_n a_n$ converges if $(na_n)_{nge1}$ is a bounded sequence, and there exist $rho,varthetage0$ and $varepsilonin(0,1]$ such that
    $$ |{p inmathbb Nmid 1le ple mmbox{ and }a_{n+p} > c_n }| le rho m^{ 1 −varepsilon} $$
    for every $m$ sufficiently large, and every $nge m^vartheta$.


  2. The series $sum_n a_n$ diverges if there exist $omega> 0$ and $lambdage0$ such that the inequality
    $$ max_{1le ple m} a_{km+p}ge frac{omega}{(km+m)^{1+lambda/m}} $$
    holds for infinitely many $m$ and every $k$.



To apply the test to the series above, one needs to know something about rational approximations to $pi$ (naturally). Actually, the authors show that to apply the test to show the divergence of the first series only requires that $pi$ is irrational, and to show the convergence of the second series only needs that $pi$ is not a Liouville number. The paper is reasonably self-contained.






share|cite|improve this answer











$endgroup$









  • 1




    $begingroup$
    Here's a screenshot of the nice elementary proof in the RMS: i.imgur.com/aGu7gze.jpg
    $endgroup$
    – Gabriel Romon
    Feb 1 at 14:49



















6












$begingroup$

see also here



http://www.siam.org/journals/categories/99-005.php




A Calculus Exam Misprint (Solved)



Summary: A misprint from a calculus exam yields a problem that possibly cannot be answered by currently known methods. Specifically, the exam question asked whether the series $sum_{n=1}^{infty} frac{(2 + sin n)^n}{3^n , n}$ converges.







share|cite|improve this answer











$endgroup$





















    3












    $begingroup$

    I propose the following heuristic argument that the series converges:



    The natural numbers $n$ are uniformly distributed ${rm mod} 2pi$. Therefore the expected value of the $n$-th term of the series is
    $$a_n:={1over n}int_{-pi}^pileft({2+cosphiover 3}right)^n dphi .$$
    Now a look at the graphs shows that
    $${2+cosphiover 3}leq e^{-phi^2/9}qquad(-pileqphileqpi) .$$
    Therefore
    $$a_nleq{1over n}int_{-pi}^pi e^{-nphi^2/9} dphi<{1over n} int_{-infty}^infty e^{-nphi^2/9} dphi={sqrt{3pi}over n^{3/2}} ,$$
    which leads to convergence.






    share|cite|improve this answer









    $endgroup$





















      1












      $begingroup$

      (incomplete proof)



      Consider this sequence:



      $$v_k = sum_{p=p_{k,min}}^{p_{k,max}} u_p$$



      where
      $p_{k,min}=[2kpi]+1$
      and $p_{k,max}=[2(k+1)pi]$
      and $u_p = frac{(sin(p)+2)^p}{p3^p}$



      1/ we have $sum_{n=1}^{infty} u_n = sum_{n=1}^{infty} v_n$



      notice that $mathbb N = cup_{k in mathbb{N}} I_k$ where $I_k=[p_{k,min},p_{k,max}]$
      and both $v_k>0$ and $u_n>0$



      2/ $v_k$ can be bounded with a convergente term



      Fact 1:
      $I_k$ can contain exactly 6 or 7 natural numbers



      Fact 2:
      each interval of the solution of $sin(x)geq 0.9$ have a lenght less than 2asin(0.9)-pi<1
      so it can't contain 2 natural numbers.



      we have two cases:



      Case 1: for every p in $I_k$ $sin(p)<0.9$
      $u_p < frac{(2,9/3)^p}{p} $



      so $v_k<7frac{(2,9/3)^p_{k,min}}{p_{k,min}} $



      Case 2: there is one p in $I_k$ such that $sin(p)geq 0.9$
      p+3 is also in $I_k$ and $sin(p+3)<0.5$



      ... this part need more thinking, i ll be back if i find something, or hope someone can use this






      share|cite|improve this answer











      $endgroup$













      • $begingroup$
        It seems that by "$IN$" you mean $mathbb N$? You can produce that with mathbb N.
        $endgroup$
        – joriki
        Feb 16 '12 at 11:18






      • 1




        $begingroup$
        Case $2$ is the hard part, so I'm afraid you haven't made any progress with this.
        $endgroup$
        – joriki
        Feb 16 '12 at 11:20










      • $begingroup$
        i just share , maybe that will help someone
        $endgroup$
        – Hassan
        Feb 16 '12 at 11:29






      • 2




        $begingroup$
        Sharing is good, of course, but it's also good to read what others have already written and take it into account. If you read the comments under the question, you'll find that several people have already put thought into how to deal with the case where $sin n$ gets close to $1$. Also, if you're already aware that you only dealt with the easy part, but you think it might be valuable to share nonetheless, it would make more sense to title it something like "an approach" or "an idea", not "incomplete proof" -- that sounds like you've made progress and the missing part is less than half the work.
        $endgroup$
        – joriki
        Feb 16 '12 at 11:44














      Your Answer





      StackExchange.ifUsing("editor", function () {
      return StackExchange.using("mathjaxEditing", function () {
      StackExchange.MarkdownEditor.creationCallbacks.add(function (editor, postfix) {
      StackExchange.mathjaxEditing.prepareWmdForMathJax(editor, postfix, [["$", "$"], ["\\(","\\)"]]);
      });
      });
      }, "mathjax-editing");

      StackExchange.ready(function() {
      var channelOptions = {
      tags: "".split(" "),
      id: "69"
      };
      initTagRenderer("".split(" "), "".split(" "), channelOptions);

      StackExchange.using("externalEditor", function() {
      // Have to fire editor after snippets, if snippets enabled
      if (StackExchange.settings.snippets.snippetsEnabled) {
      StackExchange.using("snippets", function() {
      createEditor();
      });
      }
      else {
      createEditor();
      }
      });

      function createEditor() {
      StackExchange.prepareEditor({
      heartbeatType: 'answer',
      autoActivateHeartbeat: false,
      convertImagesToLinks: true,
      noModals: true,
      showLowRepImageUploadWarning: true,
      reputationToPostImages: 10,
      bindNavPrevention: true,
      postfix: "",
      imageUploader: {
      brandingHtml: "Powered by u003ca class="icon-imgur-white" href="https://imgur.com/"u003eu003c/au003e",
      contentPolicyHtml: "User contributions licensed under u003ca href="https://creativecommons.org/licenses/by-sa/3.0/"u003ecc by-sa 3.0 with attribution requiredu003c/au003e u003ca href="https://stackoverflow.com/legal/content-policy"u003e(content policy)u003c/au003e",
      allowUrls: true
      },
      noCode: true, onDemand: true,
      discardSelector: ".discard-answer"
      ,immediatelyShowMarkdownHelp:true
      });


      }
      });














      draft saved

      draft discarded


















      StackExchange.ready(
      function () {
      StackExchange.openid.initPostLogin('.new-post-login', 'https%3a%2f%2fmath.stackexchange.com%2fquestions%2f109029%2fconvergence-divergence-of-infinite-series-sum-n-1-infty-frac-sinn2%23new-answer', 'question_page');
      }
      );

      Post as a guest















      Required, but never shown

























      5 Answers
      5






      active

      oldest

      votes








      5 Answers
      5






      active

      oldest

      votes









      active

      oldest

      votes






      active

      oldest

      votes









      28





      +100







      $begingroup$

      The values for which $sin(n)$ is close to $1$ (say in an interval $[1-varepsilon ; 1]$) are somewhat regular :



      $1 - varepsilon le sin(n)$ implies that there exists an integer $k(n)$ such that
      $n = 2k(n) pi + frac pi 2 + a(n)$ where $|a(n)| leq arccos(1- varepsilon)$.
      As $varepsilon to 0$, $arccos(1- varepsilon) sim sqrt{2 varepsilon}$, thus
      we can safely say that for $varepsilon$ small enough, $|n-2k(n) pi - frac{pi}2| = |a(n)| leq 2 sqrt{ varepsilon} $



      If $m gt n$ and $sin(n)$ and $sin(m)$ are both in $[1-varepsilon ; 1]$,
      then we have the inequality $|(m-n) - 2(k(m)-k(n)) pi| leq |m-2k(m)pi - frac{pi}2| + |n-2k(n)pi - frac{pi}2| leq 4 sqrt { varepsilon} $ where $(k(m)-k(n))$ is some integer $k$.



      Since $pi$ has a finite irrationality measure, we know that there is a finite real constant $mu gt 2$ such that for any integers $n,k$ large enough,
      $|n-k pi| ge k^{1- mu} $.



      By picking $varepsilon$ small enough we can forget about the finite number of exceptions to the inequality, and we get $ 4sqrt{varepsilon} ge (2k)^{1- mu}$.
      Thus $(m-n) ge 2kpi - 4sqrt{varepsilon} ge pi(4sqrt{varepsilon})^{frac1{1- mu}} - 4sqrt{varepsilon} ge A_varepsilon = Asqrt{varepsilon}^{frac1{1- mu}} $ for some constant $A$.



      Therefore, we have a guarantee on the lengh of the gaps between equally problematic terms, and we know how this length grows as $varepsilon$ gets smaller (as we look for more problematic terms)



      We can get a lower bound for the first problematic term using the irrationality measure as well : from $|n-2k(n) pi - frac{pi}2| leq 2sqrt {varepsilon}$, we get that for $varepsilon$ small enough, $(4k+1)^{1- mu} le |2n - (4k+1) pi| le 4sqrt varepsilon$, and then $n ge B_varepsilon = Bsqrtvarepsilon^{frac1{1- mu}}$ for some constant $B$.



      Therefore, there exists a constant $C$ such that forall $varepsilon$ small enough, the $k$-th integer $n$ such that $1-varepsilon le sin n$ is greater than $C_varepsilon k = Csqrtvarepsilon^{frac1{1- mu}}k$



      Since $varepsilon < 1$ and $frac 1 {1- mu} < 0$, this bound $C_ varepsilon$ grows when $varepsilon$ gets smaller.
      And furthermore, the speed of this growth is greater if we can pick a smaller (better) value for $mu$ (though all that matters is that $mu$ is finite)





      Now let us give an upper bound on the contribution of the terms where $n$ is an integer such that $sin (n) in [1-2varepsilon ; 1-varepsilon]$



      $$S_varepsilon = sum frac{(2+sin(n))^n}{n3^n} le sum_{kge 1} frac{(1- varepsilon/3)^{kC_{2varepsilon}}}{kC_{2varepsilon}} = frac{- log (1- (1- varepsilon/3)^{C_{2varepsilon}})}{C_{2varepsilon}} \
      le frac{- log (1- (1- C_{2varepsilon} varepsilon/3))}{C_{2varepsilon}}
      = frac{- log (C_{2varepsilon} varepsilon/3))}{C_{2varepsilon}}
      $$



      $C_{2varepsilon} = C sqrt{2varepsilon}^frac 1 {1- mu} = C' varepsilon^nu$ with $ nu = frac 1 {2(1- mu)} in ] -1/2 ; 0[$, so :



      $$ S_varepsilon le - frac{ log (C'/3) + (1+ nu) log varepsilon}{C'varepsilon^nu}
      $$





      Finally, we have to check if the series $sum S_{2^{-k}}$ converges or not :



      $$ sum S_{2^{-k}} le sum - frac { log (C'/3) - k(1+ nu) log 2}{C' 2^{-knu}}
      = sum (A+Bk)(2^ nu)^k $$



      Since $2^ nu < 1$, the series converges.






      share|cite|improve this answer











      $endgroup$













      • $begingroup$
        @ bgins : I don't think so. In order to recognize a Lebesgue-type thing, from what I remember I would have to sort the terms according to the value of ((2+sin(n))/3)^n and not simply sin(n), and it would be harder to evaluate the corresponding contribution. Maybe someone more knowledgeable can answer about this.
        $endgroup$
        – mercio
        Feb 16 '12 at 17:29






      • 1




        $begingroup$
        +1, very nice. I started down the same $sin n in [1-2varepsilon ; 1-varepsilon]$ route but didn't immediately see how to derive the required bounds that you derive in the first part -- I didn't pursue it any further when I saw Robert's comment about $sum_{n=1}^infty frac{|sin(n t)|^n}{n}$. Doesn't this proof also apply to that series for $t=1$? It seems the only differences are the factor of $1/3$ and the absolute value, but neither should matter. (By the way I think it should be $muge2$ instead of $mugt2$?)
        $endgroup$
        – joriki
        Feb 16 '12 at 18:02








      • 1




        $begingroup$
        @joriki : I think the proof applies to $sum frac{|sin(nt)|^n}n$ for any $t$ where the irrationality measure of $frac t pi$ is finite. mathworld says Salikhov proved his result in 2008, I don't know if the irrationality measure of $pi$ was proven to be finite before then or not. About $mu$, originally I wanted to be on the safe side and thought of it as a number strictly greater than the irrationality measure (which is an infimum, maybe not a minimum...). Maybe we can have $mu = 2$ in some cases but it doesn't matter ultimately.
        $endgroup$
        – mercio
        Feb 16 '12 at 18:41






      • 1




        $begingroup$
        mercio: Right. Then the appeal to Salikhov might be debatable, since the finiteness of $mu$ was known before Salikhov's explicit upper bound (am I right if I say this goes back at least (and maybe exactly) to Mahler 1953 with an estimate like $muleqslant20$?).
        $endgroup$
        – Did
        Feb 17 '12 at 12:54






      • 2




        $begingroup$
        mercio: If I may insist: hence one cannot write that Salikhov proved that $pi$ has a finite irrationality measure (fourth paragraph).
        $endgroup$
        – Did
        Feb 17 '12 at 15:10
















      28





      +100







      $begingroup$

      The values for which $sin(n)$ is close to $1$ (say in an interval $[1-varepsilon ; 1]$) are somewhat regular :



      $1 - varepsilon le sin(n)$ implies that there exists an integer $k(n)$ such that
      $n = 2k(n) pi + frac pi 2 + a(n)$ where $|a(n)| leq arccos(1- varepsilon)$.
      As $varepsilon to 0$, $arccos(1- varepsilon) sim sqrt{2 varepsilon}$, thus
      we can safely say that for $varepsilon$ small enough, $|n-2k(n) pi - frac{pi}2| = |a(n)| leq 2 sqrt{ varepsilon} $



      If $m gt n$ and $sin(n)$ and $sin(m)$ are both in $[1-varepsilon ; 1]$,
      then we have the inequality $|(m-n) - 2(k(m)-k(n)) pi| leq |m-2k(m)pi - frac{pi}2| + |n-2k(n)pi - frac{pi}2| leq 4 sqrt { varepsilon} $ where $(k(m)-k(n))$ is some integer $k$.



      Since $pi$ has a finite irrationality measure, we know that there is a finite real constant $mu gt 2$ such that for any integers $n,k$ large enough,
      $|n-k pi| ge k^{1- mu} $.



      By picking $varepsilon$ small enough we can forget about the finite number of exceptions to the inequality, and we get $ 4sqrt{varepsilon} ge (2k)^{1- mu}$.
      Thus $(m-n) ge 2kpi - 4sqrt{varepsilon} ge pi(4sqrt{varepsilon})^{frac1{1- mu}} - 4sqrt{varepsilon} ge A_varepsilon = Asqrt{varepsilon}^{frac1{1- mu}} $ for some constant $A$.



      Therefore, we have a guarantee on the lengh of the gaps between equally problematic terms, and we know how this length grows as $varepsilon$ gets smaller (as we look for more problematic terms)



      We can get a lower bound for the first problematic term using the irrationality measure as well : from $|n-2k(n) pi - frac{pi}2| leq 2sqrt {varepsilon}$, we get that for $varepsilon$ small enough, $(4k+1)^{1- mu} le |2n - (4k+1) pi| le 4sqrt varepsilon$, and then $n ge B_varepsilon = Bsqrtvarepsilon^{frac1{1- mu}}$ for some constant $B$.



      Therefore, there exists a constant $C$ such that forall $varepsilon$ small enough, the $k$-th integer $n$ such that $1-varepsilon le sin n$ is greater than $C_varepsilon k = Csqrtvarepsilon^{frac1{1- mu}}k$



      Since $varepsilon < 1$ and $frac 1 {1- mu} < 0$, this bound $C_ varepsilon$ grows when $varepsilon$ gets smaller.
      And furthermore, the speed of this growth is greater if we can pick a smaller (better) value for $mu$ (though all that matters is that $mu$ is finite)





      Now let us give an upper bound on the contribution of the terms where $n$ is an integer such that $sin (n) in [1-2varepsilon ; 1-varepsilon]$



      $$S_varepsilon = sum frac{(2+sin(n))^n}{n3^n} le sum_{kge 1} frac{(1- varepsilon/3)^{kC_{2varepsilon}}}{kC_{2varepsilon}} = frac{- log (1- (1- varepsilon/3)^{C_{2varepsilon}})}{C_{2varepsilon}} \
      le frac{- log (1- (1- C_{2varepsilon} varepsilon/3))}{C_{2varepsilon}}
      = frac{- log (C_{2varepsilon} varepsilon/3))}{C_{2varepsilon}}
      $$



      $C_{2varepsilon} = C sqrt{2varepsilon}^frac 1 {1- mu} = C' varepsilon^nu$ with $ nu = frac 1 {2(1- mu)} in ] -1/2 ; 0[$, so :



      $$ S_varepsilon le - frac{ log (C'/3) + (1+ nu) log varepsilon}{C'varepsilon^nu}
      $$





      Finally, we have to check if the series $sum S_{2^{-k}}$ converges or not :



      $$ sum S_{2^{-k}} le sum - frac { log (C'/3) - k(1+ nu) log 2}{C' 2^{-knu}}
      = sum (A+Bk)(2^ nu)^k $$



      Since $2^ nu < 1$, the series converges.






      share|cite|improve this answer











      $endgroup$













      • $begingroup$
        @ bgins : I don't think so. In order to recognize a Lebesgue-type thing, from what I remember I would have to sort the terms according to the value of ((2+sin(n))/3)^n and not simply sin(n), and it would be harder to evaluate the corresponding contribution. Maybe someone more knowledgeable can answer about this.
        $endgroup$
        – mercio
        Feb 16 '12 at 17:29






      • 1




        $begingroup$
        +1, very nice. I started down the same $sin n in [1-2varepsilon ; 1-varepsilon]$ route but didn't immediately see how to derive the required bounds that you derive in the first part -- I didn't pursue it any further when I saw Robert's comment about $sum_{n=1}^infty frac{|sin(n t)|^n}{n}$. Doesn't this proof also apply to that series for $t=1$? It seems the only differences are the factor of $1/3$ and the absolute value, but neither should matter. (By the way I think it should be $muge2$ instead of $mugt2$?)
        $endgroup$
        – joriki
        Feb 16 '12 at 18:02








      • 1




        $begingroup$
        @joriki : I think the proof applies to $sum frac{|sin(nt)|^n}n$ for any $t$ where the irrationality measure of $frac t pi$ is finite. mathworld says Salikhov proved his result in 2008, I don't know if the irrationality measure of $pi$ was proven to be finite before then or not. About $mu$, originally I wanted to be on the safe side and thought of it as a number strictly greater than the irrationality measure (which is an infimum, maybe not a minimum...). Maybe we can have $mu = 2$ in some cases but it doesn't matter ultimately.
        $endgroup$
        – mercio
        Feb 16 '12 at 18:41






      • 1




        $begingroup$
        mercio: Right. Then the appeal to Salikhov might be debatable, since the finiteness of $mu$ was known before Salikhov's explicit upper bound (am I right if I say this goes back at least (and maybe exactly) to Mahler 1953 with an estimate like $muleqslant20$?).
        $endgroup$
        – Did
        Feb 17 '12 at 12:54






      • 2




        $begingroup$
        mercio: If I may insist: hence one cannot write that Salikhov proved that $pi$ has a finite irrationality measure (fourth paragraph).
        $endgroup$
        – Did
        Feb 17 '12 at 15:10














      28





      +100







      28





      +100



      28




      +100



      $begingroup$

      The values for which $sin(n)$ is close to $1$ (say in an interval $[1-varepsilon ; 1]$) are somewhat regular :



      $1 - varepsilon le sin(n)$ implies that there exists an integer $k(n)$ such that
      $n = 2k(n) pi + frac pi 2 + a(n)$ where $|a(n)| leq arccos(1- varepsilon)$.
      As $varepsilon to 0$, $arccos(1- varepsilon) sim sqrt{2 varepsilon}$, thus
      we can safely say that for $varepsilon$ small enough, $|n-2k(n) pi - frac{pi}2| = |a(n)| leq 2 sqrt{ varepsilon} $



      If $m gt n$ and $sin(n)$ and $sin(m)$ are both in $[1-varepsilon ; 1]$,
      then we have the inequality $|(m-n) - 2(k(m)-k(n)) pi| leq |m-2k(m)pi - frac{pi}2| + |n-2k(n)pi - frac{pi}2| leq 4 sqrt { varepsilon} $ where $(k(m)-k(n))$ is some integer $k$.



      Since $pi$ has a finite irrationality measure, we know that there is a finite real constant $mu gt 2$ such that for any integers $n,k$ large enough,
      $|n-k pi| ge k^{1- mu} $.



      By picking $varepsilon$ small enough we can forget about the finite number of exceptions to the inequality, and we get $ 4sqrt{varepsilon} ge (2k)^{1- mu}$.
      Thus $(m-n) ge 2kpi - 4sqrt{varepsilon} ge pi(4sqrt{varepsilon})^{frac1{1- mu}} - 4sqrt{varepsilon} ge A_varepsilon = Asqrt{varepsilon}^{frac1{1- mu}} $ for some constant $A$.



      Therefore, we have a guarantee on the lengh of the gaps between equally problematic terms, and we know how this length grows as $varepsilon$ gets smaller (as we look for more problematic terms)



      We can get a lower bound for the first problematic term using the irrationality measure as well : from $|n-2k(n) pi - frac{pi}2| leq 2sqrt {varepsilon}$, we get that for $varepsilon$ small enough, $(4k+1)^{1- mu} le |2n - (4k+1) pi| le 4sqrt varepsilon$, and then $n ge B_varepsilon = Bsqrtvarepsilon^{frac1{1- mu}}$ for some constant $B$.



      Therefore, there exists a constant $C$ such that forall $varepsilon$ small enough, the $k$-th integer $n$ such that $1-varepsilon le sin n$ is greater than $C_varepsilon k = Csqrtvarepsilon^{frac1{1- mu}}k$



      Since $varepsilon < 1$ and $frac 1 {1- mu} < 0$, this bound $C_ varepsilon$ grows when $varepsilon$ gets smaller.
      And furthermore, the speed of this growth is greater if we can pick a smaller (better) value for $mu$ (though all that matters is that $mu$ is finite)





      Now let us give an upper bound on the contribution of the terms where $n$ is an integer such that $sin (n) in [1-2varepsilon ; 1-varepsilon]$



      $$S_varepsilon = sum frac{(2+sin(n))^n}{n3^n} le sum_{kge 1} frac{(1- varepsilon/3)^{kC_{2varepsilon}}}{kC_{2varepsilon}} = frac{- log (1- (1- varepsilon/3)^{C_{2varepsilon}})}{C_{2varepsilon}} \
      le frac{- log (1- (1- C_{2varepsilon} varepsilon/3))}{C_{2varepsilon}}
      = frac{- log (C_{2varepsilon} varepsilon/3))}{C_{2varepsilon}}
      $$



      $C_{2varepsilon} = C sqrt{2varepsilon}^frac 1 {1- mu} = C' varepsilon^nu$ with $ nu = frac 1 {2(1- mu)} in ] -1/2 ; 0[$, so :



      $$ S_varepsilon le - frac{ log (C'/3) + (1+ nu) log varepsilon}{C'varepsilon^nu}
      $$





      Finally, we have to check if the series $sum S_{2^{-k}}$ converges or not :



      $$ sum S_{2^{-k}} le sum - frac { log (C'/3) - k(1+ nu) log 2}{C' 2^{-knu}}
      = sum (A+Bk)(2^ nu)^k $$



      Since $2^ nu < 1$, the series converges.






      share|cite|improve this answer











      $endgroup$



      The values for which $sin(n)$ is close to $1$ (say in an interval $[1-varepsilon ; 1]$) are somewhat regular :



      $1 - varepsilon le sin(n)$ implies that there exists an integer $k(n)$ such that
      $n = 2k(n) pi + frac pi 2 + a(n)$ where $|a(n)| leq arccos(1- varepsilon)$.
      As $varepsilon to 0$, $arccos(1- varepsilon) sim sqrt{2 varepsilon}$, thus
      we can safely say that for $varepsilon$ small enough, $|n-2k(n) pi - frac{pi}2| = |a(n)| leq 2 sqrt{ varepsilon} $



      If $m gt n$ and $sin(n)$ and $sin(m)$ are both in $[1-varepsilon ; 1]$,
      then we have the inequality $|(m-n) - 2(k(m)-k(n)) pi| leq |m-2k(m)pi - frac{pi}2| + |n-2k(n)pi - frac{pi}2| leq 4 sqrt { varepsilon} $ where $(k(m)-k(n))$ is some integer $k$.



      Since $pi$ has a finite irrationality measure, we know that there is a finite real constant $mu gt 2$ such that for any integers $n,k$ large enough,
      $|n-k pi| ge k^{1- mu} $.



      By picking $varepsilon$ small enough we can forget about the finite number of exceptions to the inequality, and we get $ 4sqrt{varepsilon} ge (2k)^{1- mu}$.
      Thus $(m-n) ge 2kpi - 4sqrt{varepsilon} ge pi(4sqrt{varepsilon})^{frac1{1- mu}} - 4sqrt{varepsilon} ge A_varepsilon = Asqrt{varepsilon}^{frac1{1- mu}} $ for some constant $A$.



      Therefore, we have a guarantee on the lengh of the gaps between equally problematic terms, and we know how this length grows as $varepsilon$ gets smaller (as we look for more problematic terms)



      We can get a lower bound for the first problematic term using the irrationality measure as well : from $|n-2k(n) pi - frac{pi}2| leq 2sqrt {varepsilon}$, we get that for $varepsilon$ small enough, $(4k+1)^{1- mu} le |2n - (4k+1) pi| le 4sqrt varepsilon$, and then $n ge B_varepsilon = Bsqrtvarepsilon^{frac1{1- mu}}$ for some constant $B$.



      Therefore, there exists a constant $C$ such that forall $varepsilon$ small enough, the $k$-th integer $n$ such that $1-varepsilon le sin n$ is greater than $C_varepsilon k = Csqrtvarepsilon^{frac1{1- mu}}k$



      Since $varepsilon < 1$ and $frac 1 {1- mu} < 0$, this bound $C_ varepsilon$ grows when $varepsilon$ gets smaller.
      And furthermore, the speed of this growth is greater if we can pick a smaller (better) value for $mu$ (though all that matters is that $mu$ is finite)





      Now let us give an upper bound on the contribution of the terms where $n$ is an integer such that $sin (n) in [1-2varepsilon ; 1-varepsilon]$



      $$S_varepsilon = sum frac{(2+sin(n))^n}{n3^n} le sum_{kge 1} frac{(1- varepsilon/3)^{kC_{2varepsilon}}}{kC_{2varepsilon}} = frac{- log (1- (1- varepsilon/3)^{C_{2varepsilon}})}{C_{2varepsilon}} \
      le frac{- log (1- (1- C_{2varepsilon} varepsilon/3))}{C_{2varepsilon}}
      = frac{- log (C_{2varepsilon} varepsilon/3))}{C_{2varepsilon}}
      $$



      $C_{2varepsilon} = C sqrt{2varepsilon}^frac 1 {1- mu} = C' varepsilon^nu$ with $ nu = frac 1 {2(1- mu)} in ] -1/2 ; 0[$, so :



      $$ S_varepsilon le - frac{ log (C'/3) + (1+ nu) log varepsilon}{C'varepsilon^nu}
      $$





      Finally, we have to check if the series $sum S_{2^{-k}}$ converges or not :



      $$ sum S_{2^{-k}} le sum - frac { log (C'/3) - k(1+ nu) log 2}{C' 2^{-knu}}
      = sum (A+Bk)(2^ nu)^k $$



      Since $2^ nu < 1$, the series converges.







      share|cite|improve this answer














      share|cite|improve this answer



      share|cite|improve this answer








      edited Feb 17 '12 at 15:15

























      answered Feb 16 '12 at 15:46









      merciomercio

      44.8k258112




      44.8k258112












      • $begingroup$
        @ bgins : I don't think so. In order to recognize a Lebesgue-type thing, from what I remember I would have to sort the terms according to the value of ((2+sin(n))/3)^n and not simply sin(n), and it would be harder to evaluate the corresponding contribution. Maybe someone more knowledgeable can answer about this.
        $endgroup$
        – mercio
        Feb 16 '12 at 17:29






      • 1




        $begingroup$
        +1, very nice. I started down the same $sin n in [1-2varepsilon ; 1-varepsilon]$ route but didn't immediately see how to derive the required bounds that you derive in the first part -- I didn't pursue it any further when I saw Robert's comment about $sum_{n=1}^infty frac{|sin(n t)|^n}{n}$. Doesn't this proof also apply to that series for $t=1$? It seems the only differences are the factor of $1/3$ and the absolute value, but neither should matter. (By the way I think it should be $muge2$ instead of $mugt2$?)
        $endgroup$
        – joriki
        Feb 16 '12 at 18:02








      • 1




        $begingroup$
        @joriki : I think the proof applies to $sum frac{|sin(nt)|^n}n$ for any $t$ where the irrationality measure of $frac t pi$ is finite. mathworld says Salikhov proved his result in 2008, I don't know if the irrationality measure of $pi$ was proven to be finite before then or not. About $mu$, originally I wanted to be on the safe side and thought of it as a number strictly greater than the irrationality measure (which is an infimum, maybe not a minimum...). Maybe we can have $mu = 2$ in some cases but it doesn't matter ultimately.
        $endgroup$
        – mercio
        Feb 16 '12 at 18:41






      • 1




        $begingroup$
        mercio: Right. Then the appeal to Salikhov might be debatable, since the finiteness of $mu$ was known before Salikhov's explicit upper bound (am I right if I say this goes back at least (and maybe exactly) to Mahler 1953 with an estimate like $muleqslant20$?).
        $endgroup$
        – Did
        Feb 17 '12 at 12:54






      • 2




        $begingroup$
        mercio: If I may insist: hence one cannot write that Salikhov proved that $pi$ has a finite irrationality measure (fourth paragraph).
        $endgroup$
        – Did
        Feb 17 '12 at 15:10


















      • $begingroup$
        @ bgins : I don't think so. In order to recognize a Lebesgue-type thing, from what I remember I would have to sort the terms according to the value of ((2+sin(n))/3)^n and not simply sin(n), and it would be harder to evaluate the corresponding contribution. Maybe someone more knowledgeable can answer about this.
        $endgroup$
        – mercio
        Feb 16 '12 at 17:29






      • 1




        $begingroup$
        +1, very nice. I started down the same $sin n in [1-2varepsilon ; 1-varepsilon]$ route but didn't immediately see how to derive the required bounds that you derive in the first part -- I didn't pursue it any further when I saw Robert's comment about $sum_{n=1}^infty frac{|sin(n t)|^n}{n}$. Doesn't this proof also apply to that series for $t=1$? It seems the only differences are the factor of $1/3$ and the absolute value, but neither should matter. (By the way I think it should be $muge2$ instead of $mugt2$?)
        $endgroup$
        – joriki
        Feb 16 '12 at 18:02








      • 1




        $begingroup$
        @joriki : I think the proof applies to $sum frac{|sin(nt)|^n}n$ for any $t$ where the irrationality measure of $frac t pi$ is finite. mathworld says Salikhov proved his result in 2008, I don't know if the irrationality measure of $pi$ was proven to be finite before then or not. About $mu$, originally I wanted to be on the safe side and thought of it as a number strictly greater than the irrationality measure (which is an infimum, maybe not a minimum...). Maybe we can have $mu = 2$ in some cases but it doesn't matter ultimately.
        $endgroup$
        – mercio
        Feb 16 '12 at 18:41






      • 1




        $begingroup$
        mercio: Right. Then the appeal to Salikhov might be debatable, since the finiteness of $mu$ was known before Salikhov's explicit upper bound (am I right if I say this goes back at least (and maybe exactly) to Mahler 1953 with an estimate like $muleqslant20$?).
        $endgroup$
        – Did
        Feb 17 '12 at 12:54






      • 2




        $begingroup$
        mercio: If I may insist: hence one cannot write that Salikhov proved that $pi$ has a finite irrationality measure (fourth paragraph).
        $endgroup$
        – Did
        Feb 17 '12 at 15:10
















      $begingroup$
      @ bgins : I don't think so. In order to recognize a Lebesgue-type thing, from what I remember I would have to sort the terms according to the value of ((2+sin(n))/3)^n and not simply sin(n), and it would be harder to evaluate the corresponding contribution. Maybe someone more knowledgeable can answer about this.
      $endgroup$
      – mercio
      Feb 16 '12 at 17:29




      $begingroup$
      @ bgins : I don't think so. In order to recognize a Lebesgue-type thing, from what I remember I would have to sort the terms according to the value of ((2+sin(n))/3)^n and not simply sin(n), and it would be harder to evaluate the corresponding contribution. Maybe someone more knowledgeable can answer about this.
      $endgroup$
      – mercio
      Feb 16 '12 at 17:29




      1




      1




      $begingroup$
      +1, very nice. I started down the same $sin n in [1-2varepsilon ; 1-varepsilon]$ route but didn't immediately see how to derive the required bounds that you derive in the first part -- I didn't pursue it any further when I saw Robert's comment about $sum_{n=1}^infty frac{|sin(n t)|^n}{n}$. Doesn't this proof also apply to that series for $t=1$? It seems the only differences are the factor of $1/3$ and the absolute value, but neither should matter. (By the way I think it should be $muge2$ instead of $mugt2$?)
      $endgroup$
      – joriki
      Feb 16 '12 at 18:02






      $begingroup$
      +1, very nice. I started down the same $sin n in [1-2varepsilon ; 1-varepsilon]$ route but didn't immediately see how to derive the required bounds that you derive in the first part -- I didn't pursue it any further when I saw Robert's comment about $sum_{n=1}^infty frac{|sin(n t)|^n}{n}$. Doesn't this proof also apply to that series for $t=1$? It seems the only differences are the factor of $1/3$ and the absolute value, but neither should matter. (By the way I think it should be $muge2$ instead of $mugt2$?)
      $endgroup$
      – joriki
      Feb 16 '12 at 18:02






      1




      1




      $begingroup$
      @joriki : I think the proof applies to $sum frac{|sin(nt)|^n}n$ for any $t$ where the irrationality measure of $frac t pi$ is finite. mathworld says Salikhov proved his result in 2008, I don't know if the irrationality measure of $pi$ was proven to be finite before then or not. About $mu$, originally I wanted to be on the safe side and thought of it as a number strictly greater than the irrationality measure (which is an infimum, maybe not a minimum...). Maybe we can have $mu = 2$ in some cases but it doesn't matter ultimately.
      $endgroup$
      – mercio
      Feb 16 '12 at 18:41




      $begingroup$
      @joriki : I think the proof applies to $sum frac{|sin(nt)|^n}n$ for any $t$ where the irrationality measure of $frac t pi$ is finite. mathworld says Salikhov proved his result in 2008, I don't know if the irrationality measure of $pi$ was proven to be finite before then or not. About $mu$, originally I wanted to be on the safe side and thought of it as a number strictly greater than the irrationality measure (which is an infimum, maybe not a minimum...). Maybe we can have $mu = 2$ in some cases but it doesn't matter ultimately.
      $endgroup$
      – mercio
      Feb 16 '12 at 18:41




      1




      1




      $begingroup$
      mercio: Right. Then the appeal to Salikhov might be debatable, since the finiteness of $mu$ was known before Salikhov's explicit upper bound (am I right if I say this goes back at least (and maybe exactly) to Mahler 1953 with an estimate like $muleqslant20$?).
      $endgroup$
      – Did
      Feb 17 '12 at 12:54




      $begingroup$
      mercio: Right. Then the appeal to Salikhov might be debatable, since the finiteness of $mu$ was known before Salikhov's explicit upper bound (am I right if I say this goes back at least (and maybe exactly) to Mahler 1953 with an estimate like $muleqslant20$?).
      $endgroup$
      – Did
      Feb 17 '12 at 12:54




      2




      2




      $begingroup$
      mercio: If I may insist: hence one cannot write that Salikhov proved that $pi$ has a finite irrationality measure (fourth paragraph).
      $endgroup$
      – Did
      Feb 17 '12 at 15:10




      $begingroup$
      mercio: If I may insist: hence one cannot write that Salikhov proved that $pi$ has a finite irrationality measure (fourth paragraph).
      $endgroup$
      – Did
      Feb 17 '12 at 15:10











      12












      $begingroup$

      I knew this looked familiar. User Unoqualunque located the key reference. Here is a more recent reference that provides a fairly general approach:




      Enrico Laeng, Vittorino Pata. A convergence–divergence test for series of nonnegative
      terms
      , Expositiones Mathematicae 29 (4), (2011) 420–424. MR2861768 (2012m:40002).




      The authors discuss a test that does not require monotonicity of the sequence, and instead focuses on how "clustered" we find similar terms within the sequence.



      They highlight that their test applies to show that
      $$ sum_{n=1}^inftyfrac1{n^{2+cos n}} $$
      diverges, while
      $$ sum_{n=1}^inftyfrac1nleft(frac{2+cos n}3right)^n $$
      converges. They say:




      Case (i) has been recently addressed, in (Revue de la filière Mathématique (RMS) 119 (2008–2009), 3–8), where the authors give a proof that was (in their own words) at the frontier between analysis and number theory. Case (ii) apparently originated in a curious way: it was proposed in a calculus exam by mistake, and remained open for a long time thereafter. A solution was devised only ten years later (SIAM Problems and Solutions (2009)), once again by means of quite sophisticated tools.




      As is to be expected, the test is very general but a bit cumbersome to state:



      Let $(c_n)_{nge1}$ be a sequence of nonnegative terms such that $sum_n c_n<+infty$. Let $(a_n)_{nge1}$ be a series of nonnegative terms. Then:




      1. The series $sum_n a_n$ converges if $(na_n)_{nge1}$ is a bounded sequence, and there exist $rho,varthetage0$ and $varepsilonin(0,1]$ such that
        $$ |{p inmathbb Nmid 1le ple mmbox{ and }a_{n+p} > c_n }| le rho m^{ 1 −varepsilon} $$
        for every $m$ sufficiently large, and every $nge m^vartheta$.


      2. The series $sum_n a_n$ diverges if there exist $omega> 0$ and $lambdage0$ such that the inequality
        $$ max_{1le ple m} a_{km+p}ge frac{omega}{(km+m)^{1+lambda/m}} $$
        holds for infinitely many $m$ and every $k$.



      To apply the test to the series above, one needs to know something about rational approximations to $pi$ (naturally). Actually, the authors show that to apply the test to show the divergence of the first series only requires that $pi$ is irrational, and to show the convergence of the second series only needs that $pi$ is not a Liouville number. The paper is reasonably self-contained.






      share|cite|improve this answer











      $endgroup$









      • 1




        $begingroup$
        Here's a screenshot of the nice elementary proof in the RMS: i.imgur.com/aGu7gze.jpg
        $endgroup$
        – Gabriel Romon
        Feb 1 at 14:49
















      12












      $begingroup$

      I knew this looked familiar. User Unoqualunque located the key reference. Here is a more recent reference that provides a fairly general approach:




      Enrico Laeng, Vittorino Pata. A convergence–divergence test for series of nonnegative
      terms
      , Expositiones Mathematicae 29 (4), (2011) 420–424. MR2861768 (2012m:40002).




      The authors discuss a test that does not require monotonicity of the sequence, and instead focuses on how "clustered" we find similar terms within the sequence.



      They highlight that their test applies to show that
      $$ sum_{n=1}^inftyfrac1{n^{2+cos n}} $$
      diverges, while
      $$ sum_{n=1}^inftyfrac1nleft(frac{2+cos n}3right)^n $$
      converges. They say:




      Case (i) has been recently addressed, in (Revue de la filière Mathématique (RMS) 119 (2008–2009), 3–8), where the authors give a proof that was (in their own words) at the frontier between analysis and number theory. Case (ii) apparently originated in a curious way: it was proposed in a calculus exam by mistake, and remained open for a long time thereafter. A solution was devised only ten years later (SIAM Problems and Solutions (2009)), once again by means of quite sophisticated tools.




      As is to be expected, the test is very general but a bit cumbersome to state:



      Let $(c_n)_{nge1}$ be a sequence of nonnegative terms such that $sum_n c_n<+infty$. Let $(a_n)_{nge1}$ be a series of nonnegative terms. Then:




      1. The series $sum_n a_n$ converges if $(na_n)_{nge1}$ is a bounded sequence, and there exist $rho,varthetage0$ and $varepsilonin(0,1]$ such that
        $$ |{p inmathbb Nmid 1le ple mmbox{ and }a_{n+p} > c_n }| le rho m^{ 1 −varepsilon} $$
        for every $m$ sufficiently large, and every $nge m^vartheta$.


      2. The series $sum_n a_n$ diverges if there exist $omega> 0$ and $lambdage0$ such that the inequality
        $$ max_{1le ple m} a_{km+p}ge frac{omega}{(km+m)^{1+lambda/m}} $$
        holds for infinitely many $m$ and every $k$.



      To apply the test to the series above, one needs to know something about rational approximations to $pi$ (naturally). Actually, the authors show that to apply the test to show the divergence of the first series only requires that $pi$ is irrational, and to show the convergence of the second series only needs that $pi$ is not a Liouville number. The paper is reasonably self-contained.






      share|cite|improve this answer











      $endgroup$









      • 1




        $begingroup$
        Here's a screenshot of the nice elementary proof in the RMS: i.imgur.com/aGu7gze.jpg
        $endgroup$
        – Gabriel Romon
        Feb 1 at 14:49














      12












      12








      12





      $begingroup$

      I knew this looked familiar. User Unoqualunque located the key reference. Here is a more recent reference that provides a fairly general approach:




      Enrico Laeng, Vittorino Pata. A convergence–divergence test for series of nonnegative
      terms
      , Expositiones Mathematicae 29 (4), (2011) 420–424. MR2861768 (2012m:40002).




      The authors discuss a test that does not require monotonicity of the sequence, and instead focuses on how "clustered" we find similar terms within the sequence.



      They highlight that their test applies to show that
      $$ sum_{n=1}^inftyfrac1{n^{2+cos n}} $$
      diverges, while
      $$ sum_{n=1}^inftyfrac1nleft(frac{2+cos n}3right)^n $$
      converges. They say:




      Case (i) has been recently addressed, in (Revue de la filière Mathématique (RMS) 119 (2008–2009), 3–8), where the authors give a proof that was (in their own words) at the frontier between analysis and number theory. Case (ii) apparently originated in a curious way: it was proposed in a calculus exam by mistake, and remained open for a long time thereafter. A solution was devised only ten years later (SIAM Problems and Solutions (2009)), once again by means of quite sophisticated tools.




      As is to be expected, the test is very general but a bit cumbersome to state:



      Let $(c_n)_{nge1}$ be a sequence of nonnegative terms such that $sum_n c_n<+infty$. Let $(a_n)_{nge1}$ be a series of nonnegative terms. Then:




      1. The series $sum_n a_n$ converges if $(na_n)_{nge1}$ is a bounded sequence, and there exist $rho,varthetage0$ and $varepsilonin(0,1]$ such that
        $$ |{p inmathbb Nmid 1le ple mmbox{ and }a_{n+p} > c_n }| le rho m^{ 1 −varepsilon} $$
        for every $m$ sufficiently large, and every $nge m^vartheta$.


      2. The series $sum_n a_n$ diverges if there exist $omega> 0$ and $lambdage0$ such that the inequality
        $$ max_{1le ple m} a_{km+p}ge frac{omega}{(km+m)^{1+lambda/m}} $$
        holds for infinitely many $m$ and every $k$.



      To apply the test to the series above, one needs to know something about rational approximations to $pi$ (naturally). Actually, the authors show that to apply the test to show the divergence of the first series only requires that $pi$ is irrational, and to show the convergence of the second series only needs that $pi$ is not a Liouville number. The paper is reasonably self-contained.






      share|cite|improve this answer











      $endgroup$



      I knew this looked familiar. User Unoqualunque located the key reference. Here is a more recent reference that provides a fairly general approach:




      Enrico Laeng, Vittorino Pata. A convergence–divergence test for series of nonnegative
      terms
      , Expositiones Mathematicae 29 (4), (2011) 420–424. MR2861768 (2012m:40002).




      The authors discuss a test that does not require monotonicity of the sequence, and instead focuses on how "clustered" we find similar terms within the sequence.



      They highlight that their test applies to show that
      $$ sum_{n=1}^inftyfrac1{n^{2+cos n}} $$
      diverges, while
      $$ sum_{n=1}^inftyfrac1nleft(frac{2+cos n}3right)^n $$
      converges. They say:




      Case (i) has been recently addressed, in (Revue de la filière Mathématique (RMS) 119 (2008–2009), 3–8), where the authors give a proof that was (in their own words) at the frontier between analysis and number theory. Case (ii) apparently originated in a curious way: it was proposed in a calculus exam by mistake, and remained open for a long time thereafter. A solution was devised only ten years later (SIAM Problems and Solutions (2009)), once again by means of quite sophisticated tools.




      As is to be expected, the test is very general but a bit cumbersome to state:



      Let $(c_n)_{nge1}$ be a sequence of nonnegative terms such that $sum_n c_n<+infty$. Let $(a_n)_{nge1}$ be a series of nonnegative terms. Then:




      1. The series $sum_n a_n$ converges if $(na_n)_{nge1}$ is a bounded sequence, and there exist $rho,varthetage0$ and $varepsilonin(0,1]$ such that
        $$ |{p inmathbb Nmid 1le ple mmbox{ and }a_{n+p} > c_n }| le rho m^{ 1 −varepsilon} $$
        for every $m$ sufficiently large, and every $nge m^vartheta$.


      2. The series $sum_n a_n$ diverges if there exist $omega> 0$ and $lambdage0$ such that the inequality
        $$ max_{1le ple m} a_{km+p}ge frac{omega}{(km+m)^{1+lambda/m}} $$
        holds for infinitely many $m$ and every $k$.



      To apply the test to the series above, one needs to know something about rational approximations to $pi$ (naturally). Actually, the authors show that to apply the test to show the divergence of the first series only requires that $pi$ is irrational, and to show the convergence of the second series only needs that $pi$ is not a Liouville number. The paper is reasonably self-contained.







      share|cite|improve this answer














      share|cite|improve this answer



      share|cite|improve this answer








      edited Apr 13 '17 at 12:19









      Community

      1




      1










      answered Feb 21 '13 at 3:01









      Andrés E. CaicedoAndrés E. Caicedo

      65.9k8160252




      65.9k8160252








      • 1




        $begingroup$
        Here's a screenshot of the nice elementary proof in the RMS: i.imgur.com/aGu7gze.jpg
        $endgroup$
        – Gabriel Romon
        Feb 1 at 14:49














      • 1




        $begingroup$
        Here's a screenshot of the nice elementary proof in the RMS: i.imgur.com/aGu7gze.jpg
        $endgroup$
        – Gabriel Romon
        Feb 1 at 14:49








      1




      1




      $begingroup$
      Here's a screenshot of the nice elementary proof in the RMS: i.imgur.com/aGu7gze.jpg
      $endgroup$
      – Gabriel Romon
      Feb 1 at 14:49




      $begingroup$
      Here's a screenshot of the nice elementary proof in the RMS: i.imgur.com/aGu7gze.jpg
      $endgroup$
      – Gabriel Romon
      Feb 1 at 14:49











      6












      $begingroup$

      see also here



      http://www.siam.org/journals/categories/99-005.php




      A Calculus Exam Misprint (Solved)



      Summary: A misprint from a calculus exam yields a problem that possibly cannot be answered by currently known methods. Specifically, the exam question asked whether the series $sum_{n=1}^{infty} frac{(2 + sin n)^n}{3^n , n}$ converges.







      share|cite|improve this answer











      $endgroup$


















        6












        $begingroup$

        see also here



        http://www.siam.org/journals/categories/99-005.php




        A Calculus Exam Misprint (Solved)



        Summary: A misprint from a calculus exam yields a problem that possibly cannot be answered by currently known methods. Specifically, the exam question asked whether the series $sum_{n=1}^{infty} frac{(2 + sin n)^n}{3^n , n}$ converges.







        share|cite|improve this answer











        $endgroup$
















          6












          6








          6





          $begingroup$

          see also here



          http://www.siam.org/journals/categories/99-005.php




          A Calculus Exam Misprint (Solved)



          Summary: A misprint from a calculus exam yields a problem that possibly cannot be answered by currently known methods. Specifically, the exam question asked whether the series $sum_{n=1}^{infty} frac{(2 + sin n)^n}{3^n , n}$ converges.







          share|cite|improve this answer











          $endgroup$



          see also here



          http://www.siam.org/journals/categories/99-005.php




          A Calculus Exam Misprint (Solved)



          Summary: A misprint from a calculus exam yields a problem that possibly cannot be answered by currently known methods. Specifically, the exam question asked whether the series $sum_{n=1}^{infty} frac{(2 + sin n)^n}{3^n , n}$ converges.








          share|cite|improve this answer














          share|cite|improve this answer



          share|cite|improve this answer








          edited Apr 17 '12 at 16:29









          Henning Makholm

          243k17311555




          243k17311555










          answered Apr 17 '12 at 9:52









          UnoqualunqueUnoqualunque

          66144




          66144























              3












              $begingroup$

              I propose the following heuristic argument that the series converges:



              The natural numbers $n$ are uniformly distributed ${rm mod} 2pi$. Therefore the expected value of the $n$-th term of the series is
              $$a_n:={1over n}int_{-pi}^pileft({2+cosphiover 3}right)^n dphi .$$
              Now a look at the graphs shows that
              $${2+cosphiover 3}leq e^{-phi^2/9}qquad(-pileqphileqpi) .$$
              Therefore
              $$a_nleq{1over n}int_{-pi}^pi e^{-nphi^2/9} dphi<{1over n} int_{-infty}^infty e^{-nphi^2/9} dphi={sqrt{3pi}over n^{3/2}} ,$$
              which leads to convergence.






              share|cite|improve this answer









              $endgroup$


















                3












                $begingroup$

                I propose the following heuristic argument that the series converges:



                The natural numbers $n$ are uniformly distributed ${rm mod} 2pi$. Therefore the expected value of the $n$-th term of the series is
                $$a_n:={1over n}int_{-pi}^pileft({2+cosphiover 3}right)^n dphi .$$
                Now a look at the graphs shows that
                $${2+cosphiover 3}leq e^{-phi^2/9}qquad(-pileqphileqpi) .$$
                Therefore
                $$a_nleq{1over n}int_{-pi}^pi e^{-nphi^2/9} dphi<{1over n} int_{-infty}^infty e^{-nphi^2/9} dphi={sqrt{3pi}over n^{3/2}} ,$$
                which leads to convergence.






                share|cite|improve this answer









                $endgroup$
















                  3












                  3








                  3





                  $begingroup$

                  I propose the following heuristic argument that the series converges:



                  The natural numbers $n$ are uniformly distributed ${rm mod} 2pi$. Therefore the expected value of the $n$-th term of the series is
                  $$a_n:={1over n}int_{-pi}^pileft({2+cosphiover 3}right)^n dphi .$$
                  Now a look at the graphs shows that
                  $${2+cosphiover 3}leq e^{-phi^2/9}qquad(-pileqphileqpi) .$$
                  Therefore
                  $$a_nleq{1over n}int_{-pi}^pi e^{-nphi^2/9} dphi<{1over n} int_{-infty}^infty e^{-nphi^2/9} dphi={sqrt{3pi}over n^{3/2}} ,$$
                  which leads to convergence.






                  share|cite|improve this answer









                  $endgroup$



                  I propose the following heuristic argument that the series converges:



                  The natural numbers $n$ are uniformly distributed ${rm mod} 2pi$. Therefore the expected value of the $n$-th term of the series is
                  $$a_n:={1over n}int_{-pi}^pileft({2+cosphiover 3}right)^n dphi .$$
                  Now a look at the graphs shows that
                  $${2+cosphiover 3}leq e^{-phi^2/9}qquad(-pileqphileqpi) .$$
                  Therefore
                  $$a_nleq{1over n}int_{-pi}^pi e^{-nphi^2/9} dphi<{1over n} int_{-infty}^infty e^{-nphi^2/9} dphi={sqrt{3pi}over n^{3/2}} ,$$
                  which leads to convergence.







                  share|cite|improve this answer












                  share|cite|improve this answer



                  share|cite|improve this answer










                  answered Feb 17 '12 at 9:54









                  Christian BlatterChristian Blatter

                  176k8115328




                  176k8115328























                      1












                      $begingroup$

                      (incomplete proof)



                      Consider this sequence:



                      $$v_k = sum_{p=p_{k,min}}^{p_{k,max}} u_p$$



                      where
                      $p_{k,min}=[2kpi]+1$
                      and $p_{k,max}=[2(k+1)pi]$
                      and $u_p = frac{(sin(p)+2)^p}{p3^p}$



                      1/ we have $sum_{n=1}^{infty} u_n = sum_{n=1}^{infty} v_n$



                      notice that $mathbb N = cup_{k in mathbb{N}} I_k$ where $I_k=[p_{k,min},p_{k,max}]$
                      and both $v_k>0$ and $u_n>0$



                      2/ $v_k$ can be bounded with a convergente term



                      Fact 1:
                      $I_k$ can contain exactly 6 or 7 natural numbers



                      Fact 2:
                      each interval of the solution of $sin(x)geq 0.9$ have a lenght less than 2asin(0.9)-pi<1
                      so it can't contain 2 natural numbers.



                      we have two cases:



                      Case 1: for every p in $I_k$ $sin(p)<0.9$
                      $u_p < frac{(2,9/3)^p}{p} $



                      so $v_k<7frac{(2,9/3)^p_{k,min}}{p_{k,min}} $



                      Case 2: there is one p in $I_k$ such that $sin(p)geq 0.9$
                      p+3 is also in $I_k$ and $sin(p+3)<0.5$



                      ... this part need more thinking, i ll be back if i find something, or hope someone can use this






                      share|cite|improve this answer











                      $endgroup$













                      • $begingroup$
                        It seems that by "$IN$" you mean $mathbb N$? You can produce that with mathbb N.
                        $endgroup$
                        – joriki
                        Feb 16 '12 at 11:18






                      • 1




                        $begingroup$
                        Case $2$ is the hard part, so I'm afraid you haven't made any progress with this.
                        $endgroup$
                        – joriki
                        Feb 16 '12 at 11:20










                      • $begingroup$
                        i just share , maybe that will help someone
                        $endgroup$
                        – Hassan
                        Feb 16 '12 at 11:29






                      • 2




                        $begingroup$
                        Sharing is good, of course, but it's also good to read what others have already written and take it into account. If you read the comments under the question, you'll find that several people have already put thought into how to deal with the case where $sin n$ gets close to $1$. Also, if you're already aware that you only dealt with the easy part, but you think it might be valuable to share nonetheless, it would make more sense to title it something like "an approach" or "an idea", not "incomplete proof" -- that sounds like you've made progress and the missing part is less than half the work.
                        $endgroup$
                        – joriki
                        Feb 16 '12 at 11:44


















                      1












                      $begingroup$

                      (incomplete proof)



                      Consider this sequence:



                      $$v_k = sum_{p=p_{k,min}}^{p_{k,max}} u_p$$



                      where
                      $p_{k,min}=[2kpi]+1$
                      and $p_{k,max}=[2(k+1)pi]$
                      and $u_p = frac{(sin(p)+2)^p}{p3^p}$



                      1/ we have $sum_{n=1}^{infty} u_n = sum_{n=1}^{infty} v_n$



                      notice that $mathbb N = cup_{k in mathbb{N}} I_k$ where $I_k=[p_{k,min},p_{k,max}]$
                      and both $v_k>0$ and $u_n>0$



                      2/ $v_k$ can be bounded with a convergente term



                      Fact 1:
                      $I_k$ can contain exactly 6 or 7 natural numbers



                      Fact 2:
                      each interval of the solution of $sin(x)geq 0.9$ have a lenght less than 2asin(0.9)-pi<1
                      so it can't contain 2 natural numbers.



                      we have two cases:



                      Case 1: for every p in $I_k$ $sin(p)<0.9$
                      $u_p < frac{(2,9/3)^p}{p} $



                      so $v_k<7frac{(2,9/3)^p_{k,min}}{p_{k,min}} $



                      Case 2: there is one p in $I_k$ such that $sin(p)geq 0.9$
                      p+3 is also in $I_k$ and $sin(p+3)<0.5$



                      ... this part need more thinking, i ll be back if i find something, or hope someone can use this






                      share|cite|improve this answer











                      $endgroup$













                      • $begingroup$
                        It seems that by "$IN$" you mean $mathbb N$? You can produce that with mathbb N.
                        $endgroup$
                        – joriki
                        Feb 16 '12 at 11:18






                      • 1




                        $begingroup$
                        Case $2$ is the hard part, so I'm afraid you haven't made any progress with this.
                        $endgroup$
                        – joriki
                        Feb 16 '12 at 11:20










                      • $begingroup$
                        i just share , maybe that will help someone
                        $endgroup$
                        – Hassan
                        Feb 16 '12 at 11:29






                      • 2




                        $begingroup$
                        Sharing is good, of course, but it's also good to read what others have already written and take it into account. If you read the comments under the question, you'll find that several people have already put thought into how to deal with the case where $sin n$ gets close to $1$. Also, if you're already aware that you only dealt with the easy part, but you think it might be valuable to share nonetheless, it would make more sense to title it something like "an approach" or "an idea", not "incomplete proof" -- that sounds like you've made progress and the missing part is less than half the work.
                        $endgroup$
                        – joriki
                        Feb 16 '12 at 11:44
















                      1












                      1








                      1





                      $begingroup$

                      (incomplete proof)



                      Consider this sequence:



                      $$v_k = sum_{p=p_{k,min}}^{p_{k,max}} u_p$$



                      where
                      $p_{k,min}=[2kpi]+1$
                      and $p_{k,max}=[2(k+1)pi]$
                      and $u_p = frac{(sin(p)+2)^p}{p3^p}$



                      1/ we have $sum_{n=1}^{infty} u_n = sum_{n=1}^{infty} v_n$



                      notice that $mathbb N = cup_{k in mathbb{N}} I_k$ where $I_k=[p_{k,min},p_{k,max}]$
                      and both $v_k>0$ and $u_n>0$



                      2/ $v_k$ can be bounded with a convergente term



                      Fact 1:
                      $I_k$ can contain exactly 6 or 7 natural numbers



                      Fact 2:
                      each interval of the solution of $sin(x)geq 0.9$ have a lenght less than 2asin(0.9)-pi<1
                      so it can't contain 2 natural numbers.



                      we have two cases:



                      Case 1: for every p in $I_k$ $sin(p)<0.9$
                      $u_p < frac{(2,9/3)^p}{p} $



                      so $v_k<7frac{(2,9/3)^p_{k,min}}{p_{k,min}} $



                      Case 2: there is one p in $I_k$ such that $sin(p)geq 0.9$
                      p+3 is also in $I_k$ and $sin(p+3)<0.5$



                      ... this part need more thinking, i ll be back if i find something, or hope someone can use this






                      share|cite|improve this answer











                      $endgroup$



                      (incomplete proof)



                      Consider this sequence:



                      $$v_k = sum_{p=p_{k,min}}^{p_{k,max}} u_p$$



                      where
                      $p_{k,min}=[2kpi]+1$
                      and $p_{k,max}=[2(k+1)pi]$
                      and $u_p = frac{(sin(p)+2)^p}{p3^p}$



                      1/ we have $sum_{n=1}^{infty} u_n = sum_{n=1}^{infty} v_n$



                      notice that $mathbb N = cup_{k in mathbb{N}} I_k$ where $I_k=[p_{k,min},p_{k,max}]$
                      and both $v_k>0$ and $u_n>0$



                      2/ $v_k$ can be bounded with a convergente term



                      Fact 1:
                      $I_k$ can contain exactly 6 or 7 natural numbers



                      Fact 2:
                      each interval of the solution of $sin(x)geq 0.9$ have a lenght less than 2asin(0.9)-pi<1
                      so it can't contain 2 natural numbers.



                      we have two cases:



                      Case 1: for every p in $I_k$ $sin(p)<0.9$
                      $u_p < frac{(2,9/3)^p}{p} $



                      so $v_k<7frac{(2,9/3)^p_{k,min}}{p_{k,min}} $



                      Case 2: there is one p in $I_k$ such that $sin(p)geq 0.9$
                      p+3 is also in $I_k$ and $sin(p+3)<0.5$



                      ... this part need more thinking, i ll be back if i find something, or hope someone can use this







                      share|cite|improve this answer














                      share|cite|improve this answer



                      share|cite|improve this answer








                      edited Feb 17 '12 at 12:49









                      Willie Wong

                      56k10111212




                      56k10111212










                      answered Feb 16 '12 at 10:26









                      HassanHassan

                      556210




                      556210












                      • $begingroup$
                        It seems that by "$IN$" you mean $mathbb N$? You can produce that with mathbb N.
                        $endgroup$
                        – joriki
                        Feb 16 '12 at 11:18






                      • 1




                        $begingroup$
                        Case $2$ is the hard part, so I'm afraid you haven't made any progress with this.
                        $endgroup$
                        – joriki
                        Feb 16 '12 at 11:20










                      • $begingroup$
                        i just share , maybe that will help someone
                        $endgroup$
                        – Hassan
                        Feb 16 '12 at 11:29






                      • 2




                        $begingroup$
                        Sharing is good, of course, but it's also good to read what others have already written and take it into account. If you read the comments under the question, you'll find that several people have already put thought into how to deal with the case where $sin n$ gets close to $1$. Also, if you're already aware that you only dealt with the easy part, but you think it might be valuable to share nonetheless, it would make more sense to title it something like "an approach" or "an idea", not "incomplete proof" -- that sounds like you've made progress and the missing part is less than half the work.
                        $endgroup$
                        – joriki
                        Feb 16 '12 at 11:44




















                      • $begingroup$
                        It seems that by "$IN$" you mean $mathbb N$? You can produce that with mathbb N.
                        $endgroup$
                        – joriki
                        Feb 16 '12 at 11:18






                      • 1




                        $begingroup$
                        Case $2$ is the hard part, so I'm afraid you haven't made any progress with this.
                        $endgroup$
                        – joriki
                        Feb 16 '12 at 11:20










                      • $begingroup$
                        i just share , maybe that will help someone
                        $endgroup$
                        – Hassan
                        Feb 16 '12 at 11:29






                      • 2




                        $begingroup$
                        Sharing is good, of course, but it's also good to read what others have already written and take it into account. If you read the comments under the question, you'll find that several people have already put thought into how to deal with the case where $sin n$ gets close to $1$. Also, if you're already aware that you only dealt with the easy part, but you think it might be valuable to share nonetheless, it would make more sense to title it something like "an approach" or "an idea", not "incomplete proof" -- that sounds like you've made progress and the missing part is less than half the work.
                        $endgroup$
                        – joriki
                        Feb 16 '12 at 11:44


















                      $begingroup$
                      It seems that by "$IN$" you mean $mathbb N$? You can produce that with mathbb N.
                      $endgroup$
                      – joriki
                      Feb 16 '12 at 11:18




                      $begingroup$
                      It seems that by "$IN$" you mean $mathbb N$? You can produce that with mathbb N.
                      $endgroup$
                      – joriki
                      Feb 16 '12 at 11:18




                      1




                      1




                      $begingroup$
                      Case $2$ is the hard part, so I'm afraid you haven't made any progress with this.
                      $endgroup$
                      – joriki
                      Feb 16 '12 at 11:20




                      $begingroup$
                      Case $2$ is the hard part, so I'm afraid you haven't made any progress with this.
                      $endgroup$
                      – joriki
                      Feb 16 '12 at 11:20












                      $begingroup$
                      i just share , maybe that will help someone
                      $endgroup$
                      – Hassan
                      Feb 16 '12 at 11:29




                      $begingroup$
                      i just share , maybe that will help someone
                      $endgroup$
                      – Hassan
                      Feb 16 '12 at 11:29




                      2




                      2




                      $begingroup$
                      Sharing is good, of course, but it's also good to read what others have already written and take it into account. If you read the comments under the question, you'll find that several people have already put thought into how to deal with the case where $sin n$ gets close to $1$. Also, if you're already aware that you only dealt with the easy part, but you think it might be valuable to share nonetheless, it would make more sense to title it something like "an approach" or "an idea", not "incomplete proof" -- that sounds like you've made progress and the missing part is less than half the work.
                      $endgroup$
                      – joriki
                      Feb 16 '12 at 11:44






                      $begingroup$
                      Sharing is good, of course, but it's also good to read what others have already written and take it into account. If you read the comments under the question, you'll find that several people have already put thought into how to deal with the case where $sin n$ gets close to $1$. Also, if you're already aware that you only dealt with the easy part, but you think it might be valuable to share nonetheless, it would make more sense to title it something like "an approach" or "an idea", not "incomplete proof" -- that sounds like you've made progress and the missing part is less than half the work.
                      $endgroup$
                      – joriki
                      Feb 16 '12 at 11:44




















                      draft saved

                      draft discarded




















































                      Thanks for contributing an answer to Mathematics Stack Exchange!


                      • Please be sure to answer the question. Provide details and share your research!

                      But avoid



                      • Asking for help, clarification, or responding to other answers.

                      • Making statements based on opinion; back them up with references or personal experience.


                      Use MathJax to format equations. MathJax reference.


                      To learn more, see our tips on writing great answers.




                      draft saved


                      draft discarded














                      StackExchange.ready(
                      function () {
                      StackExchange.openid.initPostLogin('.new-post-login', 'https%3a%2f%2fmath.stackexchange.com%2fquestions%2f109029%2fconvergence-divergence-of-infinite-series-sum-n-1-infty-frac-sinn2%23new-answer', 'question_page');
                      }
                      );

                      Post as a guest















                      Required, but never shown





















































                      Required, but never shown














                      Required, but never shown












                      Required, but never shown







                      Required, but never shown

































                      Required, but never shown














                      Required, but never shown












                      Required, but never shown







                      Required, but never shown







                      Popular posts from this blog

                      Can a sorcerer learn a 5th-level spell early by creating spell slots using the Font of Magic feature?

                      ts Property 'filter' does not exist on type '{}'

                      mat-slide-toggle shouldn't change it's state when I click cancel in confirmation window